Sección 3.2 ejercicio 3

Sección 3.2 ejercicio 3

de Melani Scagni Larrañaga -
Número de respuestas: 1

Holaa, alguien sabe cómo realizar el ejercicio 3 ? 

En respuesta a Melani Scagni Larrañaga

Re: Sección 3.2 ejercicio 3

de Marcos Barrios -

Buenas

Te comento la parte a, para que intentes usar las mismas ideas para la parte b, pero si continuas con dudas vuelve a escribir. La idea de este ejercicio es entender en un caso concreto sumas superiores e inferiores y como acotan el área (mas allá de que no se les de ese nombre). La clave del ejercicio esta en entender los valores f(k) como área de un rectángulo.

El ejercicio comienza diciendo que la función f es monótona creciente.

Veamos la desigualdad para n  = 1, n = 2 y después pasamos al caso general. Voy a mostrar la primera desigualdad, la otra es análoga

Para n = 1 tenemos que probar que \displaystyle \sum_{k = 0}^{0} f(k) \leq \int_{0}^{1} f(t) dt es decir \displaystyle f(0) \leq \int_{0}^{1} f(t) dt \leq f(1)

Como f es monótona creciente f(0) \leq f(t) para todo t \in [0,1] es decir que el gráfico de f encierra al rectángulo de base el intervalo [0,1] en el eje x y altura f(0) y por tanto su área (\int_{0}^{1} f(t) dt) es mayor.

Para n = 2 tenemos que probar que f(0) + f(1) \leq \int_{0}^{2} f(t) dt. En esta sección, con la idea intuitiva de área tenemos que \int_{0}^{2} f(t) dt = \int_{0}^{1} f(t) dt + \int_{1}^{2} f(t) dt (esta propiedad - con al definición formal de integral - se llama aditividad respecto al intervalo).

Repitiendo el argumento anterior tenemos que f(1) \leq f(t) para todo t \in [1,2] por tanto f(1) \leq \int_{1}^{2} f(t)dt.

Podemos concluir entonces que f(0) + f(1) \leq \int_{0}\leq \int_{0}^{1} f(t) dt + \int_{1}^{2} f(t) dt = \int_{0}^{2} f(t) dt

Aquí tienes una representación gráfica de la situación

El área pintada de verde es la integral de f mientras que los rectángulos negros tienen área f(k) 

En general, para cada k, f(k) representa el área de un rectángulo de base 1 y altura f(k), mientras que para todo t en el intervalo [k,k+1] se tiene que f(k) \geq f(t) es decir la figura formada entre el eje x, f(x) y las rectas x = k, x = k+1 encierra al rectángulo de vértices opuestos (k,0) y (k+1,f(k))

Por tanto f(k) \leq \int_{k}^{k+1} f(t) dt, luego sumando desde k = 0 hasta k = n-1 tenemos que \sum_{k=0}^{n-1} f(k) \leq \int_{0}^{n} f(t)dt.

La otra desigualdad de la parte a es análoga.

Para la parte b hay que adaptar el razonamiento para rectángulos mas finos.

Si tienes dudas vuelve a escribir

Saludos